FUNÇÕES CONVEXAS





\begin{picture}(200,150)
\put(0,20){\line(1,0){170}}
\put(30,0){\line(0,1)...
...bezier(70,50)(100,55)(120,75)
\qbezier(120,75)(135,90)(145,120)
\end{picture}           

Seja $ I\subseteq \mathbb{R}$ um intervalo (limitado ou não). Uma função $ f:I\longrightarrow\mathbb{R}$ é dita convexa se tiver a seguinte propriedade: Dados dois pontos $ A$ e $ B$ no gráfico de $ f$, a corda que une estes dois pontos está sempre acima do gráfico de $ f$. Dados $ x_1<x<x_2$ em $ I$, como na figura, chamando de $ \displaystyle\,
\mu=\frac{x-x_1}{x_2-x_1}\,\rule{0.cm}{0.6cm}$, temos

$\displaystyle 0\leq \mu\leq 1 \quad , \quad
x=x_1+(x-x_1)=\hspace{1.0cm}
$

$\displaystyle \hspace{1.0cm}
=x_1+\mu\,(x_2-x_1)=(1-\mu)\,x_1+\mu\,x_2 \ ,
$


ou ainda, chamando $ \lambda=1-\mu\,$,

$\displaystyle x=\lambda\,x_1+\mu\,x_2 \quad , \quad
\lambda\geq 0 \ , \ \mu\geq 0$   e$\displaystyle \quad
\lambda+\mu=1 \ .
$

Os pontos $ C$ e $ D$ da figura têm coordenadas

$\displaystyle C=(\lambda\,x_1+\mu\,x_2,f(\lambda\,x_1+\mu\,x_2))$   e$\displaystyle \qquad
D=(\lambda\,x_1+\mu\,x_2,\lambda\,f(x_1)+\mu\,f(x_2))
$

A função $ f$ é convexa quando o ponto $ D$ está sempre acima de $ C$. Isto se expressa como

$\displaystyle \forall x_1 , x_2 \in I \quad , \quad
\forall \lambda , \mu \geq 0$   com$\displaystyle \quad
\lambda+\mu=1 \quad ,
$

vale

$\displaystyle f(\lambda\,x_1+\mu\,x_2))\leq \lambda\,f(x_1)+\mu\,f(x_2) \ .
$

Uma figura $ \mathcal{A}$ é convexa quando para quaisquer dois pontos $ A$ e $ B$ de $ \mathcal{A}$, o segmento de reta que une $ A$ e $ B$ está totalmente contido em $ \mathcal{A}$. A função $ f$ é convexa se e somente se o seu epigráfico, isto é, o conjunto

$\displaystyle \mathcal{A}=\{(x,y)\in \mathbb{R}^2\mid \
x\in I \ ,\ y\geq f(x)\}
$

que está acima de seu gráfico, é convexo. Esta é a justificativa para o nome função convexa.

Proposição 1. Uma função $ f:I\longrightarrow\mathbb{R}$, definida em um intervalo $ I$ é convexa se e somente se $ \;\forall n\geq 2\;$, $ \;\forall x_1,x_2,\ldots,x_n\in I\;$, $ \;\forall \lambda_1,\lambda_2,\ldots,\lambda_n\geq0\;$, tais que $ \;\lambda_1+\lambda_2+\cdots+\lambda_n=1\;$, tem-se

$\displaystyle f\biggl(\sum_{j=1}^n\lambda_j\,x_j\biggr)\leq
 \sum_{j=1}^n\lambda_j\,f\left(x_j\right) \ .$ (1)


Dem:
\fbox{$\Longleftarrow$} Supondo válida a condição (1) acima e tomando, em particular, $ \,n=2\,$, conclui-se que $ \,f\,$ é convexa.
\fbox{$\Longrightarrow$} Suponhamos que $ f$ é convexa. Vamos provar que vale a condição (1) para todo $ n\geq 2$ por indução em $ n\,$. Trivialmente ela vale para $ \,n=2\,$ (é a própria definição de função convexa). Suponhamos que (1) vale para um certo $ n\geq 2$. Vamos mostrar que vale também para $ n+1$.
Sejam $ \;x_1,x_2,\ldots,x_{n+1}\in I\;$ e $ \;\lambda_1,\lambda_2,\ldots,\lambda_{n+1}\geq0\;$, com $ \;\lambda_1+\lambda_2+\cdots+\lambda_{n+1}=1\;$. Podemos supor que $ \,\lambda_{n+1}\neq1\,$, pois se $ \,\lambda_{n+1}=1\,$, então $ \;\lambda_1=
 \lambda_2=\cdots=\lambda_n=0\;$ e, neste caso, a condição (1) vale trivialmente, pois se reduz a $ \,f(1\cdot x_{n+1})\leq1\cdot f(x_{n+1})\,$. Supondo, então, $ \,\lambda_{n+1}\neq1\,$, chamando

$\displaystyle y=\frac{\lambda_1\,x_1+\cdots+\lambda_n\,x_n}
{1-\lambda_{n+1}}
$

e observando que $ \;\lambda_1+\cdots+\lambda_n= 1-\lambda_{n+1}\;$ implica

$\displaystyle \frac{\lambda_1}{1-\lambda_{n+1}}+\cdots+
\frac{\lambda_n}{1-\lambda_{n+1}}=1 \ ,
$

teremos, pela convexidade de $ f$ e pela hipótese de indução,

$\displaystyle f(\lambda_1\,x_1+\cdots+\lambda_{n+1}\,x_{n+1})=
f\Bigl((1-\lambda_{n+1})\,y+\lambda_{n+1}\,x_{n+1}\Bigr)\leq $


$\displaystyle \leq (1-\lambda_{n+1})\,f(y)+\lambda_{n+1}\,f(x_{n+1})=
$


$\displaystyle =(1-\lambda_{n+1})\,f\Bigl(\frac{\lambda_1}
{1-\lambda_{n+1}}\,x...
...
\frac{\lambda_n}{1-\lambda_{n+1}}\,x_n\Bigr)
+\lambda_{n+1}\,f(x_{n+1})\leq
$


$\displaystyle \leq(1-\lambda_{n+1})\biggl[
\frac{\lambda_1}{1-\lambda_{n+1}}\,...
...\frac{\lambda_n}{1-\lambda_{n+1}}\,f(x_n)\biggr]
+\lambda_{n+1}\,f(x_{n+1})=
$


$\displaystyle =\lambda_1\,f(x_1)+\cdots+\lambda_{n+1}\,f(x_{n+1}) \ ,
$

provando que vale também para $ n+1$. Isto completa a indução.

A seguir veremos que as funções convexas são razoavelmente bem comportadas.


Proposição 2. Se $ f:I\longrightarrow\mathbb{R}$ é uma função convexa, então $ f$ é contínua em todo o ponto interior de $ I$. Em particular, se $ I$ for um intervalo aberto, $ f$ é, necessariamente, contínua.

Dem:

\begin{picture}(200,150)
\put(0,20){\line(1,0){170}}
\put(30,0){\line(0,1)...
...,74)
\dottedline(126,78)(130,76)
\dottedline(128,79)(130,78)
\end{picture}


Seja $ x_0$ um ponto interior de $ I$. Vamos provar que $ f$ é contínua em $ x_0$. Sendo $ x_0$ um ponto interior de $ I\,$, podemos escolher pontos $ x_1\in I$ à esquerda de $ x_0$ e $ x_2\in I$ à direita de $ x_0\,$. Consideremos os pontos $ \,A=\bigl(x_1,f(x_1)\bigr)\,$, $ \,P=\bigl(x_0,f(x_0)\bigr)\,$ e $ \,B=\bigl(x_2,f(x_2)\bigr)\,$, que estão no gráfico de $ f$, como na figura ao lado. Pelo fato de $ f$ ser convexa, $ P$ está abaixo (ou sobre) a reta $ \,\overline{AB}\,$. Consideremos a região hachurada $ \,\mathcal{H}\,$, limitada pelas retas $ \,\overline{AP}\,$ e $ \,\overline{PB}\,$ e pelas retas verticais $ \;x=x_1\;$ e $ \;x=x_1\;$. Se o ponto $ P$ estiver sobre a reta
$ \,\overline{AB}\,$, a região $ \,\mathcal{H}\,$ se reduz ao segmento de reta $ \,\overline{AB}\,$. Vamos provar que a porção do gráfico de $ f$ entre $ x_1$ e $ x_2$ está contida na região $ \,\mathcal{H}\,$. De fato, se o gráfico de $ f$ não ficasse dentro da região $ \,\mathcal{H}\,\rule{0.cm}{0.35cm}$,
então existiria um ponto $ C\rule{0.cm}{0.4cm}$ ou um ponto $ D$ como na figura ao lado, ou ainda pontos análogos, à direita de $ P$. Mas qualquer uma destas possibilidades iria contrariar o fato de $ f$ ser convexa. Por exemplo, um ponto como $ C$ não pode pertencer ao gráfico de $ f$, pois, se pertencesse, o ponto $ P$ estaria acima do segmento $ \,\overline{CB}\,$. Um ponto como $ D$ não pode pertencer ao gráfico de $ f$, pois, se pertencesse, o ponto $ D$ estaria acima do segmento $ \,\overline{AP}\,$. Fica assim estabelecido que o gráfico de $ f$ fica contido na região $ \,\mathcal{H}\,$. Daí segue que $ f$ é contínua no ponto $ x_0$, pois deve satisfazer uma desigualdade da forma
           \begin{picture}(200,150)
\put(0,20){\line(1,0){170}}
\put(30,0){\line(0,1)...
...\circle*{3}}
\put(71,32){\mbox{$C$}}
\put(81,85){\mbox{$D$}}
\end{picture}

$\displaystyle \lvert f(x)-f(x_0)\rvert\leq L\,\lvert x-x_0\rvert
\ \ , \qquad \forall x\in[x_1,x_2] \ ,
$

onde $ L$ é o máximo entre os valores absolutos das inclinações das retas $ \;\overline{AP}\;$ e $ \;\overline{PB}\;$.

Exemplo. A figura ao lado mostra o gráfico de uma função $ \;f:(a,b]\longrightarrow\mathbb{R}\;$ convexa, definida em um intevalo que não é aberto e descontínua no ponto $ \,b\,$, que é uma extremidade do intervalo e, portanto, não é ponto interior.
           \begin{picture}(100,70)
\put(0,10){\line(1,0){80}}
\put(10,0){\line(0,1){7...
...20)(64.5,44.5)
\put(12,1){\mbox{$a$}}
\put(63,0){\mbox{$b$}}
\end{picture}


Funções Convexas Deriváveis


Se nos restringirmos à classe das funções deriváveis, vamos obter um critério de fácil aplicabilidade para verificar se uma função é convexa. Uma função convexa não é necessariamente derivável. Por exemplo, a função $ \;f:\mathbb{R}\longrightarrow\mathbb{R}\;$, $ \,f(x)=\lvert x\rvert\,$ é convexa, mas não tem derivada no ponto $ \,x=0\,$ . Vamos ver depois que, de fato, uma função convexa não pode deixar de ter derivada em muitos pontos. Ela vai poder deixar de ter derivada no máximo em um conjunto enumerável de pontos.


Proposição 3. Seja $ \;f:I\longrightarrow\mathbb{R}\;$ uma função convexa definida em um intervalo $ I$. Dados quatro pontos $ \,x_1<x_2<x_3<x_4\,$ em $ I\,$, vale

$\displaystyle \frac{f(x_2)-f(x_1)}{x_2-x_1}\leq
\frac{f(x_4)-f(x_3)}{x_4-x_3} \ .
$


Dem:

\begin{picture}(200,150)
\put(0,20){\line(1,0){170}}
\put(30,0){\line(0,1)...
...bezier(70,50)(100,55)(120,75)
\qbezier(120,75)(135,90)(145,120)
\end{picture}           
Usando a convexidade de $ f$, temos que o ponto $ B$ está abaixo da corda $ \,\overline{AC}\,$. Logo a inclinação de $ \,\overline{AB}\,$ é menor ou igual à inclinação de $ \,\overline{AC}\,$ e esta é menor ou igual à inclinação de $ \,\overline{BC}\,$. Logo

inclinação de $ \,\overline{AB}\,$ $ \leq$ inclinação de $ \,\overline{BC}\,$.

Analogamente,

inclinação de $ \,\overline{BC}\,$ $ \leq$ inclinação de $ \,\overline{CD}\,$.

Logo
inclinação de $ \,\overline{AB}\,$ $ \leq$ inclinação de $ \,\overline{CD}\,$

e a proposição fica provada.


Proposição 4. Seja $ \;f:(a,b)\longrightarrow\mathbb{R}\;$ uma função derivável. Então $ f$ é convexa se e somente se $ f'$ é crescente (isto é, $ x_1\leq x_2 \
\Longrightarrow \ f'(x_1)\leq f'(x_2)\,$).

Dem:
\fbox{$\Longrightarrow$} Suponhamos que $ f$ seja convexa e derivável. Vamos mostrar que $ f'$ é crescente. Sejam $ \;x_1<x_3\;$ pontos de $ \,(a,b)\,$. Consideremos pontos $ x_2$ e $ x_4$ de $ \,(a,b)\,$ tais que $ \,x_1<x_2<x_3<x_4\,$. Pela Proposição 3,

$\displaystyle \frac{f(x_2)-f(x_1)}{x_2-x_1}\leq
\frac{f(x_4)-f(x_3)}{x_4-x_3} \ .
$

Fazendo $ \;x_2\longrightarrow x_1\;$ e $ \;x_4\longrightarrow x_3\;$, obtemos $ \,f'(x_1)\leq f'(x_3)\,$. provando que $ f'$ é crescente.

\fbox{$\Longleftarrow$} Suponhamos que $ f'$ seja crescente. Vamos provar que $ f$ é convexa. Sejam $ \;x_1<x_2\;$ em $ \,(a,b)\,$ e sejam $ \,\lambda,\mu>0\,$, com $ \,\lambda+\mu=1\,$. Seja

$\displaystyle L:=\lambda\,f(x_1)+\mu\,f(x_2)-
f(\lambda\,x_1+\mu\,x_2) \ .
$

Precisamos mostrar que $ \,L\geq 0\,$. Mas

$\displaystyle L=\lambda\,f(x_1)+\mu\,f(x_2)-\bigl(\lambda+\mu\bigr)
f(\lambda\,x_1+\mu\,x_2)
\hspace{0.9cm}
$


$\displaystyle \hspace{1.9cm}
=\lambda\Bigl[f(x_1)-f(\lambda\,x_1+\mu\,x_2)\Bigr]+
\mu\Bigl[f(x_2)-f(\lambda\,x_1+\mu\,x_2)\Bigr] \ .
$

Pelo Teorema da Média, existem $ \,y_1\,$ e $ \,y_2\,$, com $ \,x_1<y_1<\lambda\,x_1+\mu\,x_2<y_2<x_2\,$, tais que

$\displaystyle f(\lambda\,x_1+\mu\,x_2)-f(x_1)=\bigl(\lambda\,x_1+
\mu\,x_2-x_1\bigr)f'(y_1)=\mu\bigl(x_2-x_1\bigr)f'(y_1)
$


e

$\displaystyle f(x_2)-f(\lambda\,x_1+\mu\,x_2)=\bigl(x_2-\lambda\,x_1-
\mu\,x_2\bigr)f'(y_1)=\lambda\bigl(x_2-x_1\bigr)f'(y_2) \ .
$

Segue que

$\displaystyle L=\lambda\,\mu\bigl(x_2-x_1\bigr)\Bigl[-f'(y_1)+
f'(y_2)\Bigr]\geq 0 \ .
$


Corolário. Seja $ \;f:(a,b)\longrightarrow\mathbb{R}\;$ duas vezes derivável. Então $ f$ é convexa se e somente se $ \,f''\geq 0\,$ em $ \,(a,b)\,$.

Observação. Uma função $ \;f:I\longrightarrow\mathbb{R}\;$ é dita estritamente convexa quando, dados dois pontos $ A$ e $ B$ em seu gráfico, além da corda $ \,\overline{AB}\,$ estar acima do gráfico de $ f$, ela toca o gráfico apenas nos pontos $ A$ e $ B$. Por exemplo, uma função constante é convexa, mas não é estritamente convexa. Temos que $ f$ é estritamente convexa quando

$\displaystyle \forall x_1,x_2\in I \ , \ $    com $\displaystyle \ x_1\neq x_2 \ ,
\ \forall \lambda,\mu>0 \ , \ $    com $\displaystyle \ , \
\lambda+\mu=1 \ ,
$

vale

$\displaystyle f(\lambda\,x_1+\mu\,x_2)<\lambda\,f(x_1)+\mu\,f(x_2) \ .
$

Olhando as demonstrações acima, é fácil ver que:

1 $ ^{\underline{\rm o}}$ Se $ f$ é derivável, então $ f$ é estritamente convexa se e somente se $ f'$ é estritamente crescente ( $ \,x_1<x_2 \ \Longrightarrow \ f'(x_1)<f'(x_2)\,$).

2 $ ^{\underline{\rm o}}$ Se $ f$ é duas vezes derivável e se $ \,f''(x)>0 \ , \
\forall x\in I\,$, então $ f$ é estritamente convexa. A recíproca é falsa. Exercício: Encontre um contra-exemplo. Sugestão: tome $ f(x)=x^n$, para um $ n$ conveniente.

Exemplos de Aplicações


Exemplo 1. (Desigualdade entre a média aritmética e a média geométrica) Sejam $ \,a_1,a_2,\ldots,a_n>0\,$. Provar que

$\displaystyle \sqrt[n]{a_1\,a_2\ldots a_n}\leq
\frac{a_1+a_2+\cdots+a_n}{n} \ \ ,
$

valendo a igualdade se e somente se $ \,a_1=a_2=\cdots=a_n\,$.

Solução. Consideremos a função exponencial $ \,f:\mathbb{R}\rightarrow\mathbb{R}\,$, $ f(x)=e^x=\exp x$. Como $ \,f''(x)=e^x>0\,$, concluímos que $ f$ é convexa e até estritamente convexa. Temos, então, que

$\displaystyle \forall b_1,b_2, \ldots ,b_n \in \mathbb{R}\quad , \quad
\forall \lambda_1,\lambda_2, \ldots ,\lambda_n>0$   com$\displaystyle \quad
\lambda_1+\lambda_2+\cdots +\lambda_n=1 \ ,
$

vale

$\displaystyle \exp(\lambda_1b_1+\lambda_2b_2+\cdots +\lambda_nb_n)
\leq
\lambda_1\,e^{b_1}+\lambda_2\,e^{b_2}+\cdots
+\lambda_n\,e^{b_n} \ \ ,
$

com igualdade se e somente se $ \,b_1=b_2=\cdots=b_n\,$.

Para $ \,a_1,a_2,\ldots,a_n>0\,$, aplicando a desigualdade acima com

$\displaystyle b_j=\ln a_j$   e$\displaystyle \quad
\lambda_1=\lambda_2=\cdots=\lambda_n=\frac{1}{n}
$

temos

$\displaystyle \exp\left(\frac{\ln a_1+\ln a_2+\cdots+\ln a_n}{n}
\right)\leq
\frac{\exp\ln a_1+\exp\ln a_2+\cdots+\exp\ln a_n}{n}
\ \ ,
$

isto é,

$\displaystyle (a_1\,a_2\cdots a_n)^\frac{1}{n}\leq
\frac{a_1+a_2+\cdots a_n}{n} \ \ ,
$

com igualdade se e somente se $ \,a_1=a_2=\cdots=a_n\,$.

Exemplo 2. Usando a função $ f(x)=x\ln x$, prove que

$\displaystyle \forall a,b,x,y>0 \quad , \qquad
x\,\ln\frac{x}{a}+y\,\ln\frac{y}{b}\geq
(x+y)\,\ln\frac{x+y}{a+b} \ \ ,
$

com igualdade se e somente se $ \,\displaystyle
\frac{x}{a}=\frac{y}{b}\,$.

Solução. Para a função $ f:(0,\infty)\rightarrow\mathbb{R}$ , $ \,f(x)=x\ln x\,$, temos $ \,f'(x)=1+\ln x\,$ e $ \,f'$ é estritamente crescente. Logo $ f$ é estritamente convexa. Dados $ a,b>0$, tomando $ \,\displaystyle\lambda=\frac{a}{a+b}\,$ e $ \,\displaystyle\mu=\frac{b}{a+b}\,$, temos $ \,\lambda+\mu=1\,$, $ \,\lambda \,,\mu>0\,$. Portanto

$\displaystyle f\left(\frac{x+y}{a+b}\right)=
f\left(\frac{a}{a+b}\,\frac{x}{a}...
...}\,f\left(\frac{x}{a}\right)+
\frac{b}{a+b}\,f\left(\frac{y}{b}\right) \ \ ,
$

isto é,

$\displaystyle \frac{x+y}{a+b}\,\ln\frac{x+y}{a+b}\leq
\frac{a}{a+b}\,\frac{x}{a}\,\ln\frac{x}{a}+
\frac{b}{a+b}\,\frac{y}{b}\,\ln\frac{y}{b} \ \ ,
$

ou ainda,

$\displaystyle (x+y)\,\ln\frac{x+y}{a+b}\leq
x\,\ln\frac{x}{a}+y\,\ln\frac{y}{b} \ \ ,
$

com igualdade se e somente se $ \,\displaystyle
\frac{x}{a}=\frac{y}{b}\,$.

Exemplo 3. Sejam $ p,p'>1$ números reais satisfazendo $ \,\displaystyle
\frac{1}{p}+\frac{1}{p'}=1\,$. Mostre que para todos a, b>0

$\displaystyle a\,b\leq\frac{a^p}{\,p}+\frac{b^{\,p'}}{p'} \ \ ,$ (2)

com igualdade se e somente se $ a=b$.

OBS. Um caso particular é $ \,p=p'=
\displaystyle\frac{1}{2}$. Temos, então, a desigualdade bem conhecida

$\displaystyle a\,b\leq\frac{a^2}{2}+\frac{b^2}{2} \ \ .
$



Solução.

$\displaystyle a\,b=\exp\ln \,a\,b=\exp(\ln a+\ln b)=
\exp\left(\frac{1}{p}\ln a^p+\frac{1}{p'}\ln b^{p'}
\right) \ \ .
$

Como $ \,\displaystyle
\frac{1}{p}+\frac{1}{p'}=1\,$, usando o fato que a função exponencial é estritamente convexa, temos,

$\displaystyle a\,b\leq \frac{1}{p}
\exp\ln a^{\,p}+\frac{1}{p'}\exp\ln b^{\,p'}=
\frac{a^p}{\,p}+\frac{b^{\,p'}}{p'} \ \ ,
$

com igualdade se e somente se $ a=b$.

A desigualdade deste exemplo é importante, pois dela segue, de maneira imediata, que dados $ p,p'>1$ números reais satisfazendo $ \,\displaystyle
\frac{1}{p}+\frac{1}{p'}=1\,$ e $ \,(a_1,a_2,\ldots,a_n)\in\mathbb{R}^n\,$ e $ \,(b_1,b_2,\ldots,b_n)\in\mathbb{R}^n\,$ com os $ a_j$ e $ b_j$ não nulos, tem-se

$\displaystyle \sum_{j=1}^n\,\lvert a_j\,b_j \rvert \leq
 \frac{1}{p}\sum_{j=1}^n\,\lvert a_j\rvert^p+
 \frac{1}{p'}\sum_{j=1}^n\,\lvert b_j\rvert^{p'} \ \ .$ (3)

Como conseqüência desta última, segue a famosa

Desigualdade de Hölder. Sejam $ p,p'>1\,$ números reais satisfazendo $ \,\displaystyle
\frac{1}{p}+\frac{1}{p'}=1\,$. Para quaisquer vetores $ \,\vec{x}=(x_1,x_2,\ldots,x_n)\in\mathbb{R}^n\,$ e $ \,\vec{y}=(y_1,y_2,\ldots,y_n)\in\mathbb{R}^n\,$, vale

$\displaystyle \sum_{j=1}^n\,\lvert x_j\,y_j\rvert \leq
 \biggl(\sum_{j=1}^n\,\l...
...c{1}{p}
 \biggl(\sum_{j=1}^n\,\lvert
 y_j\rvert^{p'}\biggr)^\frac{1}{p'}
 \ \ .$ (4)

OBS. Um caso particular da Desigualdade de Hölder, correspondendo a $ p=p'=\displaystyle\frac{1}{2}$, é a Desigualdade de Cauchy-Schwarz.

Demonstração:

Dados $ \,(x_1,\dots,x_n)\in\mathbb{R}^n\,$ e $ \,(y_1,\dots,y_n)\in\mathbb{R}^n\,$, sejam

$\displaystyle A=\biggl(\sum_{j=1}^n\,\lvert x_j\rvert^p\biggr)^\frac{1}{p}$    e $\displaystyle \qquad
B=\biggl(\sum_{j=1}^n\,\lvert y_j\rvert^{p'}\biggr)^\frac{1}{p'}
\ \ .
$

Se $ \,A=0\,$, então $ \,x_j=0\,$, $ \,\forall j\,$ e (4) vale trivialmente. Da mesma forma, se $ \,B=0\,$, então (4) também vale. Podemos, então, supor que $ \,A>0\,$ e $ \,B>0\,$. Sejam

$\displaystyle a_j=\frac{x_j}{A}$    e $\displaystyle \qquad
b_j=\frac{y_j}{B} \ \ , \quad \forall j \ .
$

É fácil ver que

$\displaystyle \sum_{j=1}^n\,\lvert a_j\rvert^p=\sum_{j=1}^n\,\lvert b_j\rvert^p
=1
$

Logo, pela desigualdade (3),

$\displaystyle \sum_{j=1}^n\,\lvert a_j\,b_j\rvert\leq 1 \ .
$

Segue que

$\displaystyle \sum_{j=1}^n\,\frac{\lvert x_j\,y_j\rvert}{A\,B}\leq 1 \ ,
$

isto é,

$\displaystyle \sum_{j=1}^n\,\lvert x_j\,y_j\rvert\leq A\,B \ ,
$

que é a Desigualdade de Hölder (4).

Exemplo 4. Dados números reais $ \,\alpha_1,\alpha_2,\ldots,\alpha_n>0\,$ tais que $ \,\alpha_1+\alpha_2+\cdots+\alpha_n=1\,$, então para todos $ \,a_1,\ldots,a_n,b_1,\ldots,b_n>0\,$ tem-se

$\displaystyle a_1^{\alpha_1}\cdot\ldots\cdot a_n^{\alpha_n}+
 b_1^{\alpha_1}\cd...
...{\alpha_n}\leq
 (a_1+b_1)^{\alpha_1}\cdot\ldots\cdot 
 (a_n+b_n)^{\alpha_n} \ ,$ (5)

com igualdade se e somente se $ \displaystyle\;
\frac{b_1}{a_1}=\cdots=\frac{b_n}{a_n}\;$.

Solução.

Considere a função $ \,f:\mathbb{R}\longrightarrow\mathbb{R}\,$, $ \,f(x)=\ln\Bigl(1+e^x\Bigr)\,$. Temos

$\displaystyle f'(x)=\frac{e^x}{1+e^x} \qquad,\qquad
f''(x)=\frac{e^x\bigl(1+e^x\bigr)-e^{2\,x}}
{\bigl(1+e^x\bigr)^2}=
\frac{e^x}{\bigl(1+e^x\bigr)^2}>0 \ .
$

Logo $ f$ é estritamente convexa. Portanto,

$\displaystyle f\Bigl(\alpha_1\ln\frac{b_1}{a_1}+\cdots+
\alpha_n\ln\frac{b_n}{...
...\ln\frac{b_1}{a_1}\Bigr)+\cdots+
\alpha_nf\Bigl(\ln\frac{b_n}{a_n}\Bigr) \ ,
$

isto é,

$\displaystyle f\Bigl(\ln\Bigl[\Bigl(\frac{b_1}{a_1}\Bigr)^{\alpha_1}
\cdots\Bi...
...1+\frac{b_1}{a_1}\Bigr)+
\cdots+\alpha_n\ln\Bigl(1+\frac{b_n}{a_n}\Bigr) \ ,
$

ou ainda,

$\displaystyle \ln\biggl(1+\frac{b_1^{\alpha_1}\cdots b_n^{\alpha_n}}
{a_1^{\al...
...gr)^{\alpha_1}
\cdots\Bigl(\frac{a_n+b_n}{a_n}\Bigr)^{\alpha_n}
\biggr] \ .
$

Logo

$\displaystyle \ln\biggl(\frac{a_1^{\alpha_1}\cdot\ldots\cdot
a_n^{\alpha_n}+ b...
...n+b_n)^{\alpha_n}}{a_1^{\alpha_1}\cdot\ldots\cdot
a_n^{\alpha_n}}\biggr) \ .
$

Daí segue a desigualdade (5). Como $ f$ é estritamente convexa, ocorre igualdade se e somente se $ \displaystyle\,\ln\frac{b_1}{a_1}=\cdots=
\ln\frac{b_n}{a_n}\,$, isto é, se e somente se $ \displaystyle\,\frac{b_1}{a_1}=\cdots=\frac{b_n}{a_n}\,$.



next up previous
Next: About this document ...
Eduardo H. M. Brietzke 2001-11-12